LSAT Question Types Flashcards
The pundit’s argument requires the assumption that
Necessary Assumption
Which one of the following is an assumption required by the argument?
Pseudo sufficient assumption
Which one of the following is an assumption on which the critic’s argument relies?
necessary assumption
Which one of the following most accurately describes how the scientist’s argument proceeds?
method of reasoning or descriptive
Which one of the following, if true, would most weaken the environment minister’s argument?
weaken
Which one of the following, if true, most helps to account for the response of auto safety experts to the popularity of SUVs?
resolve, reconcile or explain
The argument in the political advertisement is most vulnerable to criticism on the grounds that it
flaw or descriptive weakening
Which one of the following most accurately expresses the main conclusion of the client’s argument?
main conclusion or main point
Which one of the following, if true, most strengthens the support for the scientists’ hypothesis?
.
strengthen
Which one of the following principles, if valid, most strongly supports the reasoning in the argument?
strengthen
Which one of the following, if true, most calls into question the police chief’s explanation for the drop in crime?
weaken
The reasoning in the commentator’s argument is most vulnerable to criticism on the grounds that the argument
flaw or descriptive weakening
Which one of the following is most supported by Dr. Khan’s statements?
most strongly supported
Which one of the following most accurately describes the role played in the argument by the claim that society would not be better off if the government enacted laws requiring people to be polite to each other?
argument part
Which one of the following, if true, would most strengthen the astronomer’s argument?
strengthen
Which one of the following most accurately describes the role played in the argument by the claim that the greatest expense in irrigated agriculture is in pumping the water?
argument part
The statements above, if true, most strongly support which one of the following?
most strongly supported
The reasoning in the police captain’s argument is most vulnerable to criticism on the grounds that the argument
flaw or descriptive weakening
Which one of the following, if true of the economist’s country, most helps to resolve the apparent paradox in the economist’s statements?
resolve, reconcile or explain
Which one of the following conclusions is most strongly supported by the information above?
most strongly supported